How Do You Calculate dy and deltay for a Polynomial Function?

  • Thread starter Thread starter fitz_calc
  • Start date Start date
  • Tags Tags
    Differential
Click For Summary
To calculate dy and deltay for the polynomial function y=2x^3-4x at x=2 with dx=0.1, dy is determined using the derivative, yielding dy=2. The correct formula for deltay involves evaluating the function at x+dx and subtracting the original function value, resulting in deltay=y(2.1)-y(2). The calculation shows that deltay equals 2.12, which is derived from the difference between the two function evaluations. Understanding the distinction between dy and deltay is crucial for accurate computation.
fitz_calc
Messages
41
Reaction score
0

Homework Statement



Find the value of dy and deltay for y=2x^3-4x, x=2 and dx = deltax = 0.1.

The Attempt at a Solution



dy/dt= 6(4)(.1)-4(.1) = 2

This is correct. However, I do not understand how to get deltay ? I tried 2/2*2^3-4(2) = .25 -- however, the answer is 2.12. how do they get this value?
 
Physics news on Phys.org
Well first thing note that delta-y is simply the change in the value of y due to delta-x.

Hence delta-y= y2 - y1

where y2 is the new value of y due to delta-x (ie. y2 = 2(x+delta-x)^3 - 4(x+delta-x))

and y1 is already given as y in the equation in the problem.
 
Last edited:
fitz_calc said:

Homework Statement



Find the value of dy and deltay for y=2x^3-4x, x=2 and dx = deltax = 0.1.

The Attempt at a Solution



dy/dt= 6(4)(.1)-4(.1) = 2[/quote
This is correct.
Actually that is far from correct, but only because it is written incorrectly! There is NO "dy/dt" because y is not a function of t. What you meant was simply
dy= (dy/dx)dx= (6x^2- 4)= (6(4)- 4)(0.1)

However, I do not understand how to get deltay ? I tried 2/2*2^3-4(2) = .25 -- however, the answer is 2.12. how do they get this value?
y(x+ deltax)= y(2.1= 2(2.1)^3- 4(2.1)
y(x)= y(2)= 2(2^3)- 4(2)

delta y is the difference of those two: deltay= f(x+deltax)- f(x).
 
Question: A clock's minute hand has length 4 and its hour hand has length 3. What is the distance between the tips at the moment when it is increasing most rapidly?(Putnam Exam Question) Answer: Making assumption that both the hands moves at constant angular velocities, the answer is ## \sqrt{7} .## But don't you think this assumption is somewhat doubtful and wrong?

Similar threads

  • · Replies 25 ·
Replies
25
Views
2K
  • · Replies 2 ·
Replies
2
Views
1K
  • · Replies 8 ·
Replies
8
Views
1K
  • · Replies 10 ·
Replies
10
Views
2K
  • · Replies 2 ·
Replies
2
Views
2K
Replies
2
Views
1K
  • · Replies 4 ·
Replies
4
Views
2K
Replies
3
Views
2K
  • · Replies 2 ·
Replies
2
Views
2K
  • · Replies 3 ·
Replies
3
Views
2K